Additional Exercises for Ch 2: Systems of Linear Equations

Review Exercises

Graphically, find the point of intersection of the two lines and That is, graph each line and see where they intersect.

You have a system of equations in two variables, . Explain the geometric significance of
(a)
No solution.
(b)
A unique solution.
(c)
An infinite number of solutions.
Consider the following augmented matrix in which denotes an arbitrary number and denotes a nonzero number. Determine whether the given augmented matrix is consistent. If consistent, is the solution unique?

The solution exists does not exist and is unique not unique .

Consider the following augmented matrix in which denotes an arbitrary number and denotes a nonzero number. Determine whether the given augmented matrix is consistent. If consistent, is the solution unique? The solution exists does not exist and is [correct]unique not unique .
Consider the following augmented matrix in which denotes an arbitrary number and denotes a nonzero number. Determine whether the given augmented matrix is consistent. If consistent, is the solution unique? The solution exists does not exist and is unique not unique .
Consider the following augmented matrix in which denotes an arbitrary number and denotes a nonzero number. Determine whether the given augmented matrix is consistent. If consistent, is the solution unique? Click the arrow to see the answer.

The third equation implies that . The fourth equation implies that . We conclude that the system is inconsistent.

Suppose a system of equations has fewer equations than variables. Will such a system necessarily be consistent? If so, explain why and if not, give an example which is not consistent.

Click the arrow to see the answer.

No. Consider and

If a system of equations has more equations than variables, can it have a solution? If so, give an example and if not, tell why not.

Click the arrow to see the answer.

These can have a solution. For example, even has an infinite set of solutions.

Find such that is the augmented matrix of an inconsistent system.

Find such that is the augmented matrix of a consistent system.

Click the arrow to see the answer.

Any will work.

Find such that is the augmented matrix of a consistent system.

Click the arrow to see the answer.

Any will work.

Choose and such that the augmented matrix shown has each of the following:
(a)
one solution
(b)
no solution
(c)
infinitely many solutions
Click the arrow to see the answer.

If there will be a unique solution for any . If and there are no solutions. If and then there are infinitely many solutions.

Choose and such that the augmented matrix shown has each of the following:
(a)
one solution
(b)
no solution
(c)
infinitely many solutions
Click the arrow to see the answer.

If then there is exactly one solution. If and then there are no solutions. If and then there are infinitely many solutions.

Determine if the system is consistent. If so, is the solution unique?

Click the arrow to see the answer.

There is no solution. The system is inconsistent. You can see this from the augmented matrix.

Determine if the system is consistent. If so, is the solution unique?

Click the arrow to see the answer.

Solution is:

Determine which matrices are in reduced row-echelon form.
(a)
yes no
(b)
yes no
(c)
yes no
Row reduce the following matrix to row echelon form. Then continue to obtain the reduced row echelon form.

Click the arrow to see the answer.

Row reduce the following matrix to row echelon form. Then continue to obtain the reduced row echelon form.

Click the arrow to see the answer.

Row reduce the following matrix to row echelon form. Then continue to obtain the reduced row echelon form.

Click the arrow to see the answer.

Row reduce the following matrix to row echelon form. Then continue to obtain the reduced row echelon form.

Click the arrow to see the answer.

Row reduce the following matrix to row echelon form. Then continue to obtain the reduced row echelon form.

Click the arrow to see the answer.

Row reduce the following matrix to row echelon form. Then continue to obtain the reduced row echelon form.

Click the arrow to see the answer.

Row reduce the following matrix to row echelon form. Then continue to obtain the reduced row echelon form.

Click the arrow to see the answer.

Find the solution of the system whose augmented matrix is

Click the arrow to see the answer.

Find the solution of the system whose augmented matrix is Click the arrow to see the answer.

The reduced row echelon form is Therefore, the solution is of the form where .

Find the solution of the system whose augmented matrix is Click the arrow to see the answer.

The reduced row echelon form is and so the solution is

Find the solution of the system whose augmented matrix is Click the arrow to see the answer.

The reduced row echelon form is and so .

Find the solution of the system whose augmented matrix is Click the arrow to see the answer.

The reduced row echelon form is . Therefore, let Then the other variables are given by

Find the solution to the system of equations, and
Find the solution to the system of equations, and
Find the solution to the system of equations, and
Find the solution to the system of equations, and
Find the solution to the system of equations, and
Find the solution to the system of equations, and
Find the solution to the system of equations, and
Find the solution to the system of equations, , , and .
Find the solution to the system of equations,
Suppose a system of equations has fewer equations than variables and you have found a solution to this system of equations. Is it possible that your solution is the only one? Explain.
No. Consider what happens when you solve the system and .
Suppose a system of linear equations has a augmented matrix and the last column is a pivot column. Could the system of linear equations be consistent? Explain.

Click the arrow to see the answer.

No. If the last column is a pivot column, then the last row looks like this: . This would lead to

Suppose the coefficient matrix of a system of equations with variables has the property that every column is a pivot column. Does it follow that the system of equations must have a solution? If so, must the solution be unique? Explain.

The solution exists does not exist and is unique not unique .

Suppose there is a unique solution to a system of linear equations. What must be true of the pivot columns in the augmented matrix?

Click the arrow to see the answer.

The last column must not be a pivot column. The remaining columns must each be pivot columns.

The steady state temperature, , of a plate solves Laplace’s equation, One way to approximate the solution is to divide the plate into a square mesh and require the temperature at each node to equal the average of the temperature at the four adjacent nodes. In the following picture, the numbers represent the observed temperature at the indicated nodes. Find the temperature at the interior nodes, indicated by and . One of the equations is .

You need
Find the rank of the following matrix. Answer:
Find the rank of the following matrix. Answer:
Find the rank of the following matrix. Answer:
Find the rank of the following matrix. Answer:
Find the rank of the following matrix. Answer:
Find the rank of the following matrix. Answer:
Find the rank of the following matrix. Answer:
Find the rank of the following matrix. Answer:
Find the rank of the following matrix. Answer:
Find the rank of the following matrix. Answer:
Find the rank of the following matrix. Answer:
Find the rank of the following matrix. Answer:
Suppose is an matrix. Explain why the rank of is always no larger than

Click the arrow to see the answer.

It is because you cannot have more leading 1’s than columns and you cannot have more leading 1’s than rows.

State whether each of the following scenarios is possible for the system of equations . If possible, describe the solution set. That is, tell whether there exists a unique solution, no solution or infinitely many solutions. Here, denotes the augmented matrix.
(a)
is a matrix, and

Click the arrow to see the answer.

Infinite solution set.

(b)
is a matrix, and

Click the arrow to see the answer.

This surely can’t happen. If you add in another column, the rank does not get smaller.

(c)
is a matrix, and

Click the arrow to see the answer.

You can’t have the rank equal 4 if you only have two columns.

(d)
is a matrix, and

Click the arrow to see the answer.

This is possible, but contains a row , making the system inconsistent.

(e)
is a matrix, and .

Click the arrow to see the answer.

In this case, there is a unique solution.

Challenge Exercises

We show in Planes in that the graph of an equation is a plane in space when not all of , , and are zero.
(a)
By examining the possible positions of planes in space, show that three equations in three variables can have zero, one, or infinitely many solutions.
(b)
Can two equations in three variables have a unique solution? Give reasons for your answer.
Find all solutions to the following systems of linear equations.
(a)

Click the arrow to see the answer.

, , ,

(b)

In each case find (if possible) conditions on , , and such that the system has zero, one, or infinitely many solutions.
(a)

Click the arrow to see the answer.

If , no solution. If , , , . If and , the unique solution is , ,

(b)

Show that any two rows of a matrix can be interchanged by using the other two elementary row operations. (This shows that one of the elementary row operations is “redundant”.)

Click the arrow to see the answer.

If , show that has reduced row-echelon form .
Find , , and so that the system has the solution , , .

, ,

Solve the system where .
Show that the real system has a complex solution: , , where . Explain. What happens when such a real system has a unique solution?

Click the arrow to see the answer.

The (real) solution is , , where is a parameter. The given complex solution occurs when is complex. If the real system has a unique solution, that solution is real because the coefficients and constants are all real.

A man is ordered by his doctor to take units of vitamin A, units of vitamin B, and units of vitamin C each day. Three brands of vitamin pills are available, and the number of units of each vitamin per pill are shown in the accompanying table.

(a)
Find all combinations of pills that provide exactly the required amount of vitamins (no partial pills allowed).
(b)
If brands 1, 2, and 3 cost 3 cents, 2 cents, and 5 cents  per pill, respectively, find the least expensive treatment.

of brand 1, of brand 2, of brand 3

A restaurant owner plans to use tables seating , tables seating , and tables seating , for a total of tables. When fully occupied, the tables seat customers. If only half of the tables, half of the tables, and one-fourth of the tables are used, each fully occupied, then customers will be seated. Find , , and .
(a)
Show that a matrix with two rows and two columns that is in reduced row-echelon form must have one of the following forms:

Click the arrow to see the answer.

The leading in the first row must be in column 1 or 2 or not exist.

(b)
List the seven reduced row-echelon forms for matrices with two rows and three columns.
(c)
List the four reduced row-echelon forms for matrices with three rows and two columns.
An amusement park charges $ for adults, $ for youths, and $ for children. If people enter and pay a total of $, find the numbers of adults, youths, and children.
These numbers are nonnegative integers.
Solve the following system of equations for and .
These equations are linear in the new variables , , and .

Octave Exercises

Use Octave to check your work on Problems prb:2.18 to prb:2.38. The command is an easy way to obtain the reduced row echelon form of a matrix. It is more challenging to go through the steps of Gaussian elimination, but it can be done.

To use Octave, go to the Sage Math Cell Webpage, copy the code below into the cell, select OCTAVE as the language, and press EVALUATE. The code does the first two steps of prb:2.18. See if you can add one more command which will put the matrix into row echelon form.

% Solving systems using Gaussian elimination
 
A=[2 -1 3 -1; 1 0 2 1; 1 -1 1 -2];  
R=rref(A)  
A  
A(2,:)=-1/2*A(1,:)+A(2,:)  
A(3,:)=-1/2*A(1,:)+A(3,:)
Use Octave to check your work on Problems prb:2.44 to prb:2.54. The command is an easy way to obtain the rank of a matrix. Of course, you can also use and you obtain even more information! Problem prb:2.44 is entered in the code below.

To use Octave, go to the Sage Math Cell Webpage, copy the code below into the cell, select OCTAVE as the language, and press EVALUATE.

% Computing the rank of a matrix
                                                                  

                                                                  
 
A=[4 -16 -1 -5; 1 -4 0 -1; 1 -4 -1 -2]  
r=rank(A)  
R=rref(A)

Bibliography

The Review Exercises come from the end of Chapter 1 of Ken Kuttler’s A First Course in Linear Algebra. (CC-BY)

Ken Kuttler, A First Course in Linear Algebra, Lyryx 2017, Open Edition, pp. 42–49.

The Challenge Exercises come from the end of Chapter 1 of Keith Nicholson’s Linear Algebra with Applications. (CC-BY-NC-SA)

W. Keith Nicholson, Linear Algebra with Applications, Lyryx 2018, Open Edition, pp. 33–34.